준연접층: 두 판 사이의 차이

내용 삭제됨 내용 추가됨
87번째 줄:
* {{웹 인용|url=http://mathoverflow.net/questions/39941/does-qcohx-admit-a-generating-set|제목=
Does Qcoh(X) admit a generating set?|출판사=Math Overflow|언어=en}}
* {{웹 인용|url=https://math.stackexchange.com/questions/467197/examples-of-mathcalo-x-modules-that-are-not-quasi-coherent-sheaves | 제목=Examples of 𝒪<mathsub>\mathcal O_X''X''</mathsub>-modules that are not quasi-coherent sheaves | 웹사이트=Stack Exchange | 언어=en}}
 
[[분류:스킴 이론]]